Proof of Convergence of Matrix Exponential Series

  • Thread starter Thread starter rayman123
  • Start date Start date
  • Tags Tags
    Exponential Matrix
Click For Summary
SUMMARY

The discussion centers on the proof of convergence for the matrix exponential series, specifically the series defined by \(\sum_{k=0}^{+\infty}{\frac{\|A^k\|}{k!}}\). User Micromass suggests using D'Alembert's Ratio test to check for convergence, while also proposing a comparison with the series expansion of \(e^{\|A\|}\). The conclusion reached is that the series converges absolutely, confirming the validity of the proposed methods.

PREREQUISITES
  • Understanding of matrix norms and properties
  • Familiarity with series convergence tests, particularly D'Alembert's Ratio test
  • Knowledge of matrix exponential functions
  • Basic principles of mathematical analysis
NEXT STEPS
  • Study the application of D'Alembert's Ratio test in series convergence
  • Explore the properties of matrix norms in detail
  • Learn about the series expansion of exponential functions, particularly \(e^{\|A\|}\)
  • Investigate other convergence tests applicable to matrix series
USEFUL FOR

Mathematicians, students studying linear algebra, and researchers interested in the convergence properties of matrix functions.

rayman123
Messages
138
Reaction score
0

Homework Statement


I am searching for a proof of convergence of matrix exponential series. Where I can find it?
Thanks!



Homework Equations





The Attempt at a Solution

 
Physics news on Phys.org
Can you show that the series converges absolutely? I.e. that

\sum_{k=0}^{+\infty}{\frac{\|A^k\|}{k!}}

converges...
 
@Micromass : As a sidenote, can this expression be checked for convergence/divergence using D'Alembert's Ratio test?
 
Probably yes, but I prefer to compare it with the series expansion of e^{\|A\|}...
 
I found the whole proof
 
Question: A clock's minute hand has length 4 and its hour hand has length 3. What is the distance between the tips at the moment when it is increasing most rapidly?(Putnam Exam Question) Answer: Making assumption that both the hands moves at constant angular velocities, the answer is ## \sqrt{7} .## But don't you think this assumption is somewhat doubtful and wrong?

Similar threads

Replies
2
Views
1K
Replies
7
Views
2K
  • · Replies 4 ·
Replies
4
Views
1K
  • · Replies 4 ·
Replies
4
Views
1K
  • · Replies 5 ·
Replies
5
Views
2K
  • · Replies 3 ·
Replies
3
Views
1K
  • · Replies 8 ·
Replies
8
Views
3K
Replies
15
Views
2K
Replies
26
Views
2K
  • · Replies 8 ·
Replies
8
Views
2K